K
Khách

Hãy nhập câu hỏi của bạn vào đây, nếu là tài khoản VIP, bạn sẽ được ưu tiên trả lời.

27 tháng 8 2017

tuổi con HN là :

50 : ( 1 + 4 ) = 10 ( tuổi )

tuổi bố HN là :

50 - 10 = 40 ( tuổi )

hiệu của hai bố con ko thay đổi nên hiệu vẫn là 30 tuổi

ta có sơ đồ : bố : |----|----|----|

                  con : |----| hiệu 30 tuổi

tuổi con khi đó là :

 30 : ( 3 - 1 ) = 15 ( tuổi )

số năm mà bố gấp 3 tuổi con là :

 15 - 10 = 5 ( năm )

       ĐS : 5 năm

mình nha

27 tháng 8 2017

tuổi con HN là :

50 : ( 1 + 4 ) = 10 ( tuổi )

tuổi bố HN là :

50 - 10 = 40 ( tuổi )

hiệu của hai bố con ko thay đổi nên hiệu vẫn là 30 tuổi

ta có sơ đồ : bố : |----|----|----|

                  con : |----| hiệu 30 tuổi

tuổi con khi đó là :

 30 : ( 3 - 1 ) = 15 ( tuổi )

số năm mà bố gấp 3 tuổi con là :

 15 - 10 = 5 ( năm )

       ĐS : 5 năm

mình nha

6 tháng 10 2019

\(VT=\frac{ab+bc+ca}{ab}+\frac{ab+bc+ca}{bc}+\frac{ab+bc+ca}{ca}\)

\(=3+\frac{c\left(a+b\right)}{ab}+\frac{a\left(b+c\right)}{bc}+\frac{b\left(c+a\right)}{ca}\)(1)

Theo BĐT AM-GM: \(\frac{1}{2}\left[\frac{c\left(a+b\right)}{ab}+\frac{a\left(b+c\right)}{bc}\right]\ge\sqrt{\frac{\left(a+b\right)\left(b+c\right)}{b^2}}\)

Tương tự: \(\frac{1}{2}\left[\frac{a\left(b+c\right)}{bc}+\frac{b\left(c+a\right)}{ca}\right]\ge\sqrt{\frac{\left(a+c\right)\left(b+c\right)}{c^2}}\)

\(\frac{1}{2}\left[\frac{c\left(a+b\right)}{ab}+\frac{b\left(c+a\right)}{ca}\right]\ge\sqrt{\frac{\left(a+c\right)\left(a+b\right)}{a^2}}\)

Cộng theo vế 3 BĐT trên rồi thay vào 1 ta sẽ thu được đpcm.

6 tháng 10 2019

Ý em là thay vào (1) !!

27 tháng 5 2020

Bài 2:b) \(9=\left(\frac{1}{a^3}+1+1\right)+\left(\frac{1}{b^3}+1+1\right)+\left(\frac{1}{c^3}+1+1\right)\)

\(\ge3\left(\frac{1}{a}+\frac{1}{b}+\frac{1}{c}\right)\therefore\frac{1}{a}+\frac{1}{b}+\frac{1}{c}\le3\)

Ta sẽ chứng minh \(P\le\frac{1}{48}\left(\frac{1}{a}+\frac{1}{b}+\frac{1}{c}\right)^2\)

Ai có cách hay?

27 tháng 5 2020

1/Đặt a=1/x,b=1/y,c=1/z ->x+y+z=1.

2a) \(VT=\frac{\left(\frac{1}{a^3}+\frac{1}{b^3}\right)\left(\frac{1}{a}+\frac{1}{b}\right)}{\frac{1}{a}+\frac{1}{b}}\ge\frac{\left(\frac{1}{a^2}+\frac{1}{b^2}\right)^2}{\frac{1}{a}+\frac{1}{b}}\)

\(=\frac{\left[\frac{\left(a^2+b^2\right)^2}{a^4b^4}\right]}{\frac{a+b}{ab}}=\frac{\left(a^2+b^2\right)^2}{a^3b^3\left(a+b\right)}\ge\frac{\left(a+b\right)^3}{4\left(ab\right)^3}\)

\(\ge\frac{\left(a+b\right)^3}{4\left[\frac{\left(a+b\right)^2}{4}\right]^3}=\frac{16}{\left(a+b\right)^3}\)

11 tháng 11 2019

2/ Không mất tính tổng quát, giả sử \(c=min\left\{a,b,c\right\}\).

Nếu abc = 0 thì có ít nhất một số bằng 0. Giả sử c = 0. BĐT quy về: \(a^2+b^2\ge2ab\Leftrightarrow\left(a-b\right)^2\ge0\) (luôn đúng)

Đẳng thức xảy ra khi a = b; c = 0.

Nếu \(abc\ne0\). Chia hai vế của BĐT cho \(\sqrt[3]{\left(abc\right)^2}\)

BĐT quy về: \(\Sigma_{cyc}\sqrt[3]{\frac{a^4}{b^2c^2}}+3\ge2\Sigma_{cyc}\sqrt[3]{\frac{ab}{c^2}}\)

Đặt \(\sqrt[3]{\frac{a^2}{bc}}=x;\sqrt[3]{\frac{b^2}{ca}}=y;\sqrt[3]{\frac{c^2}{ab}}=z\Rightarrow xyz=1\)

Cần chúng minh: \(x^2+y^2+z^2+3\ge2\left(\frac{1}{x}+\frac{1}{y}+\frac{1}{z}\right)\)

\(\Leftrightarrow x^2+y^2+z^2+2xyz+1\ge2\left(xy+yz+zx\right)\) (1)

Theo nguyên lí Dirichlet thì trong 3 số x - 1, y - 1, z - 1 tồn tại ít nhất 2 số có tích không âm. Không mất tính tổng quát, giả sử \(\left(x-1\right)\left(y-1\right)\ge0\)

\(\Rightarrow2xyz\ge2xz+2yz-2z\). Thay vào (1):

\(VT\ge x^2+y^2+z^2+2xz+2yz-2z+1\)

\(=\left(x-y\right)^2+\left(z-1\right)^2+2xy+2xz+2yz\)

\(\ge2\left(xy+yz+zx\right)\)

Vậy (1) đúng. BĐT đã được chứng minh.

Đẳng thức xảy ra khi a = b = c hoặc a = b, c = 0 và các hoán vị.

Check giúp em vs @Nguyễn Việt Lâm, bài dài quá:(

6 tháng 7 2020

Để đưa về chứng minh $(1)$ và $(2)$ ta dùng:

Định lí SOS: Nếu \(X+Y+Z=0\) thì \(AX^2+BY^2+CZ^2\ge0\)

khi \(\left\{{}\begin{matrix}A+B+C\ge0\\AB+BC+CA\ge0\end{matrix}\right.\)

Chứng minh: Vì \(\sum\left(A+C\right)=2\left(A+B+C\right)\ge0\)

Nên ta có thể giả sử \(A+C\ge0\). Mà $X+Y+Z=0$ nên$:$

\(AX^2+BY^2+CZ^2=AX^2+BY^2+C\left[-\left(X+Y\right)\right]^2\)

\(={\frac { \left( AX+CX+CY \right) ^{2}}{A+C}}+{\frac {{Y}^{2} \left( AB+AC+BC \right) }{A+C}} \geq 0\)